Write an algebraic expression with the constant 7 and the variable 4.

Answers

Answer 1

Answer:

y = 4x + 7

Step-by-step explanation:

y = 4x + 7

7 is the constant and 4 is the coefficient

Answer 2
4x + 7. the coefficient is 7.

Related Questions

Refer to your Expeditions in Reading book for a complete version of this text.

Barton is remembered for her contributions during the Civil War.

Which three details should be included in a summary of Barton’s contributions?


Barton earns permission to join the army and provides supplies to soldiers in the field when supply wagons cannot get through.

She speaks in cities around the country to talk about the Red Cross and to encourage others to become involved.

She works with doctors treating the wounded on the front lines and expects to be treated the same as male soldiers.

Barton convinces the army to set up aid stations near every battlefield so wounded soldiers can receive immediate medical care.

Answers

Answer:

The 3rd one, The 1st one and The last one.

Step-by-step explanation:

Answer: She works with the doctors treating the wounded on the front lines and expects to be treated like the same as the male soldiers.

Barton convinces the army to set up aid stations near every battlefield so wounded soldiers can receive immediate medical care. Barton earns permission to join the army and provides supplies to soldiers in the field when supply wagons cannot get through.

Step-by-step explanation: i took the test

​What is the best estimate for this sum?

1/4+2/7


The sum will be close to 1/4.

The sum will be close to 1/2.

The sum will be close to 1.

Answers

Answer:

The sum is close to 1/2

Step-by-step explanation:

1/4 = 7/28
2/7 = 8/28
7/28 + 8/28 = 15/28

14/28 = 1/2 and 15/28 is close to 14/28

The best estimate for this sum is 1/2

This is because because if we make the expression into a decimal it would be 0.53571428.. and many other numbers. If you make 1/4 into a decimal it will be 0.25 which is not close to the other number. Where-as if we made 1/2 into a decimal it would be 0.5 which would be the closest. Hope this helps

DONT SCAM ME + BRAINLIST FOR RIGHT ANWSER
Light ray 1 strikes the smooth surface, separating air and water as shown below.

A horizontal line is drawn and the part above the line is labeled Air and the part below the line is labeled Water. There is one slanting ray labeled 1 which falls on the line at a point and there are four rays of light labeled 2, 3, 4, and 5 which move away from same point on the line. Ray 5 is exactly vertical to the line and below the line, ray 4 is on the right of ray 5 and makes an angle of about 25 degrees with ray 5. Ray 3 is above the line and makes an angle of about 25 degrees with the line. Ray 2 is on the right of ray 1 and ray 1 and ray 2 make angles of about 25 degrees with the vertical at the point where ray 1 strikes.

Which of the following rays is the part of ray 1 that is reflected from the smooth surface?

Ray 2
Ray 3
Ray 4
Ray 5

Answers

Answer:

Ray 2 is the answer

Ray 2 because it’s the smoothest

Help!!!!!!! image below.

Answers

Yes


I think it’s right if not sorry
Use the rectangle formula then implement The numbers

Which expression is equivalent to –3(y–6)?

Answers

Solution:

It should be noted:

To simplify the distributive property, multiply the outside term with all the terms inside the bracket.

Simplifying the distributive property.

-3(y - 6)=> (-3 x y) + (-3 x -6)=> -3y + 18 (Option C)

Answer:

The last one, (-6y) times -3, the reason is because its just this expression but backwards

Edit this is wrong its C my bad, i read it so wrong

HELP ASAP!!! WILL GIVE BRAINLIEST

Answers

Answer:

the answer should be 27

Step-by-step explanation:

27 is the correct answer for this equation

I need y'all help so bad

Answers

Answer:

30, 15, 50, 20

Step-by-step explanation:

I gotchu

median is found by doing this;

(see first image)

So, for data set A, it would be:

*Btw im assuming the numbers are the same on both data sets since there is the same amount of lines in each and the same space btwn the lines*

10,10,20,20,20,30,30,30,30,40,40,50

Median is: 30

Iqr is found by doing this:

(see second image)

So for data set A, it would be:

IQR:15

---------------------------------------------------------

Set B(same thing)

Median:

30,40,40,40,50,50,50,50,60,60,70,70

Median is: 50

IQR: 20

That should be all! Let me know if I'm correct ;)

6/5-4/15 can you help me get the answer?

Answers

14/15 should be the answer :) if you can find a way to simplify it I would and I’m sorry if I’m wrong
6/5 - 4/15
First let’s look at the denominator
We need to get both denominations to be the same number by using multiplication
Do you know how to do that

I’d you have this question:

A line graph shows quantity demanded on the x-axis from 0 to 30 and price in dollars from 0 to 25. A line labeled D starts at point (5,25) and decreases to point (25,5). Other points appear between. A line labeled D subscript 1 starts at point (7,22.5) and decreases to point (27,2.5). Other points appear between. A line labeled D subscript 2 is not visible, but there is an arrow pointing left for this line. An arrow points right for line D subscript 1. If the demand curve shown above shifts to the right then the __________. A. price increased B. demand increased C. price decreased D. demand decreased

The answer is B

Answers

Answer:

yes it is b tysm!

Step-by-step explanation:

Answer:

it is not b

Step-by-step explanation:

An image of a rectangular pyramid is shown below: A right rectangular pyramid is shown. Part A: A cross section of the rectangular pyramid is cut with a plane parallel to the base. What is the name of the shape created by the cross section? Explain your answer. (5 points) Part B: If a cross section of the rectangular pyramid is cut perpendicular to the base, passing through the top vertex, what would be the shape of the resulting cross section? Explain your answer. (5 points) Source StylesFormatFontSize

Answers

Part A: rectangle,
Part B: a triangle.

DO ALL OF THESE AND HELP ASAP FOR BRAINLIEST AND POINTS.

Answers

Answer:

2. n/8 3. w-8 4. 1 + q 5. 3w

6. the sum of m and 83 7. 42 groups of s 8. k less than 5 9. h divided by 12

10. 107 11. 5.3^5

Step-by-step explanation:

Answer:

maybe someone can check my answers

2. n/8

3. w-8

4. 1+q=s

5. w/3

6. the sum of m and 83

7. 42 times s

8. 5 less than k

9. 12 groups of h

10. i got 107 ??

11. 5.3^4

Which is the BEST definition of dilation?
A. a transformation that spins an object in the coordinate plane

B. a transformation that either reduces or enlarges an object in the coordinate plane

C. a transformation that flips an object in the coordinate plane

D. a transformation that either slides or shifts an object in the coordinate plane

Answers

Answer:

B

Step-by-step explanation:

The definition of dilation is the size of an object/ shape by increasing/ decreasing its dimensions by scaling factors

Answer :B. a transformation that either reduces or enlarges an object in the coordinate plane

Step-by-step explanation: di·la·tion is the action or condition of becoming or being made wider, larger, or more open.

HURRY
Which ordered pair is not a solution of the linear equation shown?
y= 1/2x


A. (2, 1)
B. (1, 1/2)
C. (4, 8)
D. (–2, –1)

Answers

c is the answer to this equation
It’s B as all others are correct

help me please please please

Answers

the answer should be 48814

Answer:

172,438

Step-by-step explanation:

The number of total doctors should be a bigger number than the number of female doctors.

Let x = the total number of doctors

x • 28.3% = 48800

Change the % to a decimal for the calculations.

.283x = 48800

Divide by .283

x = 172438

The total number of registered doctors is 172,438

PLEASE HELP DUE DATE IS 12 AND ITS 10!!!!

Answers

Answer:

nah

Step-by-step explanation:figure it out ur self

Arrange these 5 numbers in order from smallest to largest

π, 22/7, 3.14, 355/113, 3 141/1000

Answers

Answer:

[tex]\frac{335}{113} < 3.14 < 3\frac{141}{1000} < \pi[/tex]

Step-by-step explanation:

Set of Data: [tex]\pi[/tex], [tex]3.14[/tex], [tex]\frac{335}{113}[/tex], [tex]3\frac{141}{1000}[/tex]

To order them from smallest to largest:

[tex]\pi = 3.1415[/tex][tex]3.14 = 3.14[/tex][tex]\frac{335}{113} = 2.965[/tex][tex]3\frac{141}{1000} = 3.141[/tex]

Hence, the order should be:

[tex]\frac{335}{113} < 3.14 < 3\frac{141}{1000} < \pi[/tex]

The length of two sides of triangle are 4cm and 5cm. Between what two
measures should the length of the third side fall?

Answers

Answer:

1 and 9

Step-by-step explanation:

By the Triangle Inequality Theorem, the sum of two side lengths of a triangle is always greater than the length of the third side.

In other words, in a triangle with side lengths [tex]a, b,[/tex] and [tex]c,[/tex] we always have [tex]a+b > c.[/tex]

Applying this to this question, the other side length, [tex]x,[/tex] must satisfy the following inequalities:

[tex]4+5 > x,\\x+5 > 4,\\x+4 > 5.[/tex]

Solving these inequalities gives

[tex]x < 9,\\x > -1,\\x > 1.[/tex]

Combining these solutions, we have [tex]1 < x < 9.[/tex] Therefore, the length of the third side falls between 1 and 9.

The speed of 10 cars on a certain highway stretch was recorded below. If the mean of the data is 63.2 with a standard deviation of 6.2, circle all data values with a z-score less than 0.5.
{49, 58, 60, 63, 64, 64, 65, 67, 70, 72}

Answers

Answer:

58

Step-by-step explanation:

58 is the answer.

16.2 is what percent of 90?

18%

14.6%

90%

16%

Answers

Answer:

18%

Step-by-step explanation:

Step 1: We make the assumption that 90 is 100% since it is our output value.

Step 2: We next represent the value we seek with x

Step 3: From step 1, it follows that 100%=90

Step 4: In the same vein, x%=16.2

Step 5: This gives us a pair of simple equations:

Step 6: By simply dividing equation 1 by equation 2 and taking note of the fact that both the LHS

(left hand side) of both equations have the same unit (%); we have

Step 7: Taking the inverse (or reciprocal) of both sides yields

Therefore, 16.2 is 18% of 90

Answer:

16%

Step-by-step explanation: If its 16.2 out of 90 its %16 bc the .2 is not more than 5

Solve: 80 ≥ -8(w - 8)

ANSWER CHOICES:
1. w ≥ -2

2. w > -2

3. w < -2

4. w ≤ -2

Answers

Answer: w>-2

Step-by-step explanation

W>-2 is the best answer for this question

e. Giving brainliest to first [correct] answer!!

Answers

Answer:

Step-by-step explanation:

Keep, Change, Flip. Keep the 4. Change the division to multiplication. Flip the fraction, 1/5 becomes 5. Then multiply. 4 * 5 = 20.

What is a good estimate for this angle measurement?

A. 10 degrees
B. 25 degrees
C. 75 degrees
D. 95 degrees

Answers

C. 75 degreesssssss:)
C. 75 Degrees

The angle we are measuring is part of a right angle, and option D is more than 90 degrees so that rules that out. Options A and B are way too small to have such a wide angle so the only feasible option is option C

website sells popular books in hardcover, paperback, and digital form. The paperback version of a book sells for 12 dollars less than the hardcover version, and the digital version sells for half the price of the paperback copy.

The website adds a finance charge equal to 1/10 of the price of the hardcover version to all purchases.

Which expressions represent the total cost of the digital version of a book if the hardcover price is h dollars?

Answers

Answer:

The answer is

H - P - 12  D 1/2 x .10 times H

(please help asap, i'll give brainlyist to the most helpful answer. I need the whole problems answered.)
Solve using the standard algorithm.
1. 7,124÷26= 2. 8,961÷29=

3. 9,168÷24= 4. 9,120÷12=

Answers

1. 7,124÷26= 274

2. 8,961÷29= 309

3. 9,168÷24= 382

4. 9,120÷12= 760

help with these 3 questions..

Answers

#1

Lies in 1st and image in 4th quadrant.

Total rotation quadrant=4-1=3

Rotation angle=3(90)=270° and 360-270=90°

Reflection

#2

Total rotation=3-1=2

Rotation angle=2(90)=180°

Symmetrical or linear

#3

One quadrant shift

Rotation angle=90°Translational

pls help i will give brainliest

Answers

Answer:

If you multiply by 2 then it would become -16 < 4 which is 2 times as much as the original.

If you multiply by -2 then it will become 16 < -4 which is not true.

Step-by-step explanation:

If you multiply by 2 then it would become -16 < 4 which is 2 times as much as the original.

If you multiply by -2 then it will become 16 < -4 which is not true.

Answer:

when you multiply/divide/add/ subtract to both sides they'd both be equal

they prob won't have the exact same numbers but iykyk

like 1+1=2

multiply both sides by 6

when u add it'll be true

srry if it was confusing

i suck at explaining

Step-by-step explanation:

Pls help due tomorrow

Answers

Answer:

32 + 63 = 95

180(all triangle angles add up to 180) - 95 = 85

Step-by-step explanation:

Check it yourself. It’s pretty easy. :)

95 because you add them together 32+63 =95

Please help

The formula to find the volume of a cube can be written as V = s to the 3rd power.
The length of the side of a square of one cube is 5 in. what is the total volume?

Answers

Answer:

125 cubic inches

Step-by-step explanation:

5x5x5=25×5=125

Answer:

125 in cubed

Step-by-step explanation:

5 to the 3rd power is 125

A teacher surveyed her class after they had taken a vocabulary test. Eighteen of the students claimed they had studied at least one hour for the test. The remaining twelve students admitted that they had not studied for the test at all. The test results (expressed as a percent) for the two groups are shown below.

Studied: 88, 100, 94, 79, 92, 100, 95, 83, 89, 99, 100, 91, 89, 95, 100, 93, 96, 84

Did Not Study: 82, 72, 45, 91, 58, 83, 65, 87, 90, 77, 73, 89

Calculate the range and interquartile range for each set of data.
Using complete sentences, compare the data sets based on your results in part A.

ANSWER TO QUESTION

Part A

Studied : 79, 83, 84, 88, 89, 89, 91, 92, 93, 94, 95, 95, 96, 99, 100, 100, 100, 100
Range : 21
Interquartile range : 99.5 - 89 = 10.5

Did not study : 45, 58, 65, 72, 73, 77, 82, 83, 87, 89, 90, 91
Range : 46
Interquartile range : 89 - 65 = 24

Part B

The students that claimed to study had a lower range and interquartile than the students that did not study. Range difference between the students that did not study vs the students that did study, 46 > 21. Interquartile range difference between the students that did not study vs the students that did, 24 > 10.5.

Answers

Answer:

I think you're correct

Step-by-step explanation:

i dont know what you want us to di

HELP BRAINLIEST PLUS 25 POINTS
A. (2, 6) and (3, 0)
B. (6, 2) and 3, 0)
C. (6, 2) and (0, 0)
D. (2, 6) and (0, 0

Answers

B is correct
Pls mark me as the Brainliest!!!ty
Have a good day :)

Answer:

B

Step-by-step explanation:

Slope formula:

[tex]\mathsf{slope=\dfrac{(y_2-y_1)}{(x_2-x_1)}}[/tex]

where [tex]\mathsf{(x_1,y_1)}[/tex] and [tex]\mathsf{(x_2,y_2)}[/tex] are points on the line

A.

[tex]\mathsf{slope=\dfrac{(0-6)}{(3-2)}=-6}[/tex]

B.

[tex]\mathsf{slope=\dfrac{(0-2)}{(3-6)}=\dfrac23}[/tex]

C.

[tex]\mathsf{slope=\dfrac{(0-2)}{(0-6)}=\dfrac13}[/tex]

D.

[tex]\mathsf{slope=\dfrac{(0-6)}{(0-2)}=3}[/tex]

Other Questions
one of the safest and easiest investments for young people is give three health risk factors and describe one A given volume of methane diffuses in 20 seconds. How long will it take the same volume of hydrogen to diffuse under the same conditions? [CH4 = 16; H2 = 2] find the value of x trigonometry What did president eisenhower do when the arkansas governor tried to prevent black students from entering central high school?. 4^2+4^6 solve this please 43. The nurse aide will never be a part of the interdisciplinary team solve the equation -3(2x - 1)= 4x - 7 ADDITIONAL ACTIVITY (P. 15)Directions: Conduct an interview with your family members about The gradual implementation of face-to-face classes in low-risk areas. Write down their responses in a clean sheet of paper and determine the claims that they used in their arguments. Use the space provided below. Select the correct answer.In the figure, a pair of parallel lines is cut by a transversal. What kind of angle are angles A and C?A. corresponding anglesB. alternate interior anglesC. vertical anglesD. supplementary angles Please helpScience....Thanks An engineer finds that a large wind turbine for generating energy rotates at a constant angular speed of 2. 0 rad/sec. Through what angle does it rotate in one minute? what is the meaning of 31178??? Ira believes in the approach to psychology known as structuralism. How is this BEST demonstrated?A. He explores a patients unconscious mind to find the root cause of their issues.B. He organizes information based on the five senses and what they perceive.C. He involves psychological research and observation in his exploration of the mind.D. He uses introspection to identify conscious feelings and mental images. Explain what elements shouldn appear in formal writing style The __________ per capita is the most widely accepted way of measuring wealth by countries in the world today because the statistic tries to account for both income earned by residents within a country as well as income earned outside of the country that flows back in to its residents. A rectangular park has a perimeter of 374 feet and a length of 65 feet. What is the width of the park? PLEASE HELP ASAP THIS IS DUE TOMORROW AT 8:00 am AND I HAVE TO STOP WORKING ON IT IN LIKE 10 MINUTES-The Confederacy Surrenders at Appomattox: Text 11. Draw Inferences The text states, He [Grant] knew that the Union could replace men and supplies. The South could not. How did this knowledge affect Grants actions at Petersburg? Can someone help me out??? Once points are on your driving record, they will stay there for at least.